La estrategia que tiene no es la más rigurosa (por qué es $\phi$ bien definida ¿por qué? $\phi$ un morfismo de grupo). Yo sugeriría empezar con :
$$\psi:G\rightarrow G/H_1\times \dots\times G/H_n $$
$$g\mapsto (gH_1,\dots, gH_n) $$
Se trata de un morfismo de grupo, que será factorizado por $H_1\cap\dots \cap H_n$ y esto demostrará al mismo tiempo (y con rigor) que $\phi$ está bien definida, es un morfismo de grupo y es unívoca.
Claramente $Ker(\psi)=H_1\cap\dots\cap H_n$ . De hecho, $\psi(g)$ es trivial si para todo $1\leq i\leq n$ tenemos $gH_i=H_i$ si para todo $1\leq i\leq n$ $g\in H_i$ .
Por lo tanto, $\psi$ factores a través de $H_1\cap\dots\cap H_n$ por :
$$\phi:G/H_1\cap\dots \cap H_n\rightarrow G/H_1\times \dots\times G/H_n $$
$$g H_1\cap \dots \cap H_n\mapsto (gH_1,\dots, gH_n) $$
Hay que tener en cuenta que no he utilizado el hecho de que los índices sean coprimos entre sí. Por supuesto, lo utilizaremos para demostrar que $\phi$ está en. Denotemos $d_i:=[G:H_i]$ . Denote $H:=H_1\cap\dots \cap H_n$ .
Obsérvese que :
$$[G:H]=[G:H_i][H_i:H]=d_i[H_i:H]$$
Por lo tanto, $d_i$ divide $[G:H]$ para todos $i$ ya que son coprimas por pares, su producto también divide a $[G:H]$ . Diciendo $d:=d_1\dots d_n$ conseguimos que :
$$d\text{ divides } |G/H| $$
Pero $G/H$ es isomorfo a $Im(\phi)$ incluido en $G/H_1\times\dots G/H_n$ que es de cardenal $d_1\times\dots\times d_n=d$ así que $|Im(\phi)|=d$ y $\phi$ está en.
0 votos
Ver también math.stackexchange.com/questions/254448/ para una formulación diferente.
1 votos
Ver también mathoverflow.net/questions/18893/ .
0 votos
Pero, ¿por qué necesitamos que los índices de los subgrupos normales sean coprimos?